Finding and Proving Invertability of 2x2 Matrices

  • Thread starter Thread starter daniel_i_l
  • Start date Start date
  • Tags Tags
    Matrices
Click For Summary
The discussion focuses on finding 2x2 matrices A such that AA = A, identifying that these matrices act as projection operators. It is noted that if A is invertible, the only solution is A = I, while non-invertible cases correspond to one-dimensional subspaces of R^2. For proving the invertibility of A under the condition I + a1A + a2A^2 + ... + akA^k = 0, it is established that rearranging leads to a determinant condition indicating A is invertible. The conversation also suggests using a specific form of A to derive conditions for the matrix elements. Overall, the key takeaway is the relationship between the matrix's properties and its geometric interpretation as a projection.
daniel_i_l
Gold Member
Messages
864
Reaction score
0

Homework Statement


a) find all the 2x2 matrices where AA=A.
b) prove that if
<br /> I + a_{1}A + a_{2}A^2 + ... + a_{k}A^k = 0<br />
then A is invertable


Homework Equations


1)det(A) = 0 iff A isn't invertable


The Attempt at a Solution


a) I'm not sure how to approch this. I found that if A is invertable then the only solution is A=I but how do i cover the other cases?

b) by rearanging:
<br /> A(a_{1}I + a_{2}A^1 + ... + a_{k}A^{k-1}) = -I <br />
and if i take the determinant on each side i see that |A| <> 0 so it's invertable. Is that correct?
Thanks.
 
Last edited:
Physics news on Phys.org
b) looks correct.

Regarding a), assume A is regular and see what follows from that. Further on, consider A = I as a special case.
 
AA=A says that A is a projection operator. So consider what the range of A is. The 'other' case you are after is where it is a one-dimensional subspace of R^2. BTW, you don't have to show det(A) is non-zero, you've constructed an explicit inverse.
 
You could also just set A=[[a,b],[c,d]] and write the condition A^2=A. You can then eliminate two of the variables (except for singular cases). There are a two parameter family of these babies.
 
Question: A clock's minute hand has length 4 and its hour hand has length 3. What is the distance between the tips at the moment when it is increasing most rapidly?(Putnam Exam Question) Answer: Making assumption that both the hands moves at constant angular velocities, the answer is ## \sqrt{7} .## But don't you think this assumption is somewhat doubtful and wrong?

Similar threads

  • · Replies 4 ·
Replies
4
Views
2K
  • · Replies 14 ·
Replies
14
Views
2K
Replies
11
Views
2K
  • · Replies 3 ·
Replies
3
Views
1K
  • · Replies 6 ·
Replies
6
Views
2K
  • · Replies 2 ·
Replies
2
Views
1K
Replies
5
Views
3K
  • · Replies 11 ·
Replies
11
Views
2K
  • · Replies 8 ·
Replies
8
Views
2K
  • · Replies 3 ·
Replies
3
Views
2K